登录
OEIS由OEIS基金会的许多慷慨捐赠者.

 

标志
提示
(来自的问候整数序列在线百科全书!)
A094405号 a(1)=1;a(n)=(先前条款之和)mod n。
1, 1, 2, 0, 4, 2, 3, 5, 0, 8, 4, 6, 10, 4, 5, 7, 11, 1, 17, 11, 18, 10, 15, 1, 21, 11, 16, 26, 17, 27, 16, 24, 7, 5, 1, 29, 13, 17, 25, 1, 33, 15, 20, 30, 5, 45, 33, 7, 2, 42, 22, 32, 52, 38, 8, 2, 47, 23, 32, 50, 25, 35, 55, 31, 46, 10, 3, 57, 29, 41, 65, 41, 64, 36, 53, 11, 2, 62, 26 (列表;图表;参考;;历史;文本;内部格式)
抵消
1,3
评论
定理。对于n>=397的所有值,a(n)=97。证明。让s(n)表示总和[a(i),i=1..n-1]。计算表明,s(397)=38606=397*97+97。因此,a(397)=397*97+97 mod 397=97。那么s(398)=s(397)+97=398*97+97,得出a(398。一个简单的归纳参数表明,对于所有整数k>=0,A(397+k)=97-约翰·莱曼2004年6月7日
猜想:对于任何种子a(1),序列“a(n)=(前项之和)mod n”以重复常数结尾。对于a(1)=1,。。。,941中-扎克·塞多夫2006年2月24日
基本上与A066910号.[来自R.J.马塔尔2008年9月5日]
链接
例子
a(4)=0,因为前面的项1、1、2和为4,而4模4为0。a(5)=4,因为前面的项1、1、2、0和为4,4模5为4。
MAPLE公司
L:=[1];s:=1;p:=2;而(nops(L)<90)do;如果1>0,则;t:=s模块p;L:=[op(L),t];s:=s+t;p:=p+1;fi;od;L;
交叉参考
关键词
非n
作者
Chuck Seggelin(seqfan(AT)plasteddragon.com),2004年6月3日
状态
经核准的

查找|欢迎光临|维基|注册|音乐|地块2|演示|索引|浏览|更多|网络摄像头
贡献新序列。或评论|格式|样式表|转换|超级搜索|最近
OEIS社区|维护人员OEIS基金会。

许可协议、使用条款、隐私政策。.

上次修改时间:美国东部夏令时2024年3月28日09:04。包含371240个序列。(在oeis4上运行。)